0 Daumen
225 Aufrufe

Aufgabe:

Zeige fur alle n ∈ ℕ mit n ≥ 1:
1/2 \( \sum\limits_{k=1}^{n-1}{\frac{1}{k}} \) ≤ log(n) ≤ \( \sum\limits_{k=1}^{n-1}{\frac{1}{k}} \) .

Avatar von

2 Antworten

0 Daumen

Es gilt $$ \ln(n) = \int_1^n \frac{1}{x} dx $$ Berechne die Ober- und Untersummen. Die Obersumme ist $$ O(n) = \sum_{k=1}^{n-1} \frac{1}{k} $$ bei einer Diskretisierung mit \( \Delta = 1 \).

Die Untersumme ist $$ U(n) = \sum_{k=2}^{n} \frac{1}{k}  $$ mit der gleichen Diskretisierung.

Jetzt musst Du noch Nachweisen das die Untersumme größer als der angegebene Term ist. Am besten mit Induktion.

Avatar von 39 k
0 Daumen

Hallo,$$ \log(n)=\int\limits_{1}^{n}{\dfrac{1}{x} \, dx}=\int\limits_{1}^{2}{\dfrac{1}{x}\, dx}+\int\limits_{2}^{3}{\dfrac{1}{x}\, dx}+\cdots +\int\limits_{n-1}^{n}{\dfrac{1}{x}\, dx} \quad (*)$$ Für riemann-integrierbare Funktionen \(f\) gilt, insofern \(m\leq f(x)\leq M\) für alle \(x\in [a,b]\), dass $$m(b-a)\le\int\limits_{a}^{b}{f(x) dx} \le M(b-a) $$ Konkret für \(\int\limits_{1}^{n}{\dfrac{1}{x} \, dx}\) gilt damit $$\dfrac{1}{m}\le\int\limits_{m-1}^{m}{\dfrac{1}{x}\, dx}\le \dfrac{1}{m-1} \quad (\#)$$ Mit \((*)\) und \((\#)\) folgt, dass \(\sum_{k=2}^n \frac{1}{k}\leq \log(n) \leq \sum_{k=1}^{n-1} \frac{1}{k}\)

Avatar von 28 k

Ein anderes Problem?

Stell deine Frage

Willkommen bei der Mathelounge! Stell deine Frage einfach und kostenlos

x
Made by a lovely community